Problem 1! IMO 2015

We say that a finite set \(\mathcal{S}\) of points in the plane is balanced if, for any two different points A and B in S\mathcal{S}, there is a point C in S\mathcal{S} such that AC=BCAC=BC. We say that S\mathcal{S} is centre-free if for any three different points A, B and C in S\mathcal{S}, there is no points P in S\mathcal{S} such that PA=PB=PCPA=PB=PC

(a) Show that for all integers n3n\geq 3, there exists a balanced set consisting of nn points.

(b) Determine all integers n3n\geq 3 for which there exists a balanced centre-free set consisting of points.

This is part of the set IMO 2015
#Combinatorics #CombinatorialGeometry #InternationalMathOlympiad(IMO) #IMO #IMO2015

Note by Sualeh Asif
5 years, 11 months ago

No vote yet
1 vote

  Easy Math Editor

This discussion board is a place to discuss our Daily Challenges and the math and science related to those challenges. Explanations are more than just a solution — they should explain the steps and thinking strategies that you used to obtain the solution. Comments should further the discussion of math and science.

When posting on Brilliant:

  • Use the emojis to react to an explanation, whether you're congratulating a job well done , or just really confused .
  • Ask specific questions about the challenge or the steps in somebody's explanation. Well-posed questions can add a lot to the discussion, but posting "I don't understand!" doesn't help anyone.
  • Try to contribute something new to the discussion, whether it is an extension, generalization or other idea related to the challenge.
  • Stay on topic — we're all here to learn more about math and science, not to hear about your favorite get-rich-quick scheme or current world events.

MarkdownAppears as
*italics* or _italics_ italics
**bold** or __bold__ bold

- bulleted
- list

  • bulleted
  • list

1. numbered
2. list

  1. numbered
  2. list
Note: you must add a full line of space before and after lists for them to show up correctly
paragraph 1

paragraph 2

paragraph 1

paragraph 2

[example link](https://brilliant.org)example link
> This is a quote
This is a quote
    # I indented these lines
    # 4 spaces, and now they show
    # up as a code block.

    print "hello world"
# I indented these lines
# 4 spaces, and now they show
# up as a code block.

print "hello world"
MathAppears as
Remember to wrap math in \( ... \) or \[ ... \] to ensure proper formatting.
2 \times 3 2×3 2 \times 3
2^{34} 234 2^{34}
a_{i-1} ai1 a_{i-1}
\frac{2}{3} 23 \frac{2}{3}
\sqrt{2} 2 \sqrt{2}
\sum_{i=1}^3 i=13 \sum_{i=1}^3
\sin \theta sinθ \sin \theta
\boxed{123} 123 \boxed{123}

Comments

a) Consider the set of points O,P1,P2,PnO, P_1, P_2, \ldots P_n such that PkO=rP_kO=r for all 1kn1\le k\le n; and for all PiP_i, there exists a PjP_j such that PiPj=rP_iP_j=r. It is easy to construct this for any n3n\ge 3.

b) I claim that only odd integers n3n\ge 3 satisfy that it can be center-free. For all odd integers n3n\ge 3, a working example is just the vertices of a regular nn-gon. It remains to prove that it is not possible for even numbers n3n\ge 3.

Note that there are (n2)=n(n1)2\dbinom{n}{2} = \dfrac{n(n-1)}{2} edges total, and nn points, meaning that on average, each point lies on the perpendicular bisector of n12\dfrac{n-1}{2} edges. When nn is even, this means that there exists a point PP that lies on the perpendicular bisector of n2\dfrac{n}{2} edges. Consider the edges that have PP on their perpendicular bisector. At least two edges share a point, because if no edge shared a point there can be at most n21\dfrac{n}{2}-1 edges as there are only n1n-1 available points. But then we're done; since two edges share a point, and PP is a perpendicular bisector of both these edges, then the distance between PP to the three points that make up these two edges is the same, contradiction.

Daniel Liu - 5 years, 11 months ago

We can prove (b) using double counting(refer to Daniel's solution for the rest), here's the basic outline:

Our object being counted will be (P,{A,B})(P, \{ A,B\}) satisfying PA=PBPA=PB, the number of which will be denoted by kk. On one hand, every combination of two points(i.e. {A,B}\{A,B\}) has at least one PP since the set is balanced; this gives kC(n,2)=n(n1)2k\ge C(n,2)=\frac {n(n-1)}{2}. On the other hand, every point has at most n12\lfloor \frac {n-1}{2}\rfloor combinations of two points (If it has more, then by PHP there exists two combinations that share a point, contradicting the centre-free condition); this gives knn12k\le n\lfloor \frac {n-1}{2}\rfloor. Together we have:

n(n1)2nn12 \frac {n(n-1)}{2}\le n\lfloor \frac {n-1}{2}\rfloor

Clearly this inequality only holds when nn is odd.

Xuming Liang - 5 years, 11 months ago
×

Problem Loading...

Note Loading...

Set Loading...